Erfährt Licht eine Doppler-Verschiebung entlang und gegen Frame-Dragging?

Stellen Sie sich 2 Photonen vor, die sich in entgegengesetzte Richtungen entlang des Äquators bewegen. Ich schlage natürlich nicht vor, dass Licht in die Umlaufbahn gelangt. Ich wandere nur, ob diese Photonen blau und rot verschoben werden, so leicht wie ein massiver Körper wie die Erddrehung in der allgemeinen Relativitätstheorie?

Ich schlage nicht vor, dass Licht in die Umlaufbahn geht - warum nicht? Die Photonenkugel ist ein Ding...
Siehe die richtige Antwort hier: physical.stackexchange.com/questions/457995/…

Antworten (2)

Um dies zu beantworten, müssen wir mit der Gleichung beginnen, die die Geometrie um eine kugelsymmetrische rotierende Masse herum beschreibt. Dies ist die Kerr-Metrik. Ich werde dies unten vollständig aufschreiben, was ziemlich beängstigend aussehen wird, aber für unsere Zwecke finden wir, dass die Gleichung viel vereinfacht:

D S 2 = ( 1 R S R ρ 2 ) D T 2 + ρ 2 Δ D R 2 + ρ 2 D θ 2 + ( R 2 + a 2 + R S R a 2 ρ 2 Sünde 2 θ ) Sünde 2 θ D ϕ 2 + 2 R S R a Sünde 2 θ ρ 2 D T D ϕ

Wo:

R S = 2 M a = J M ρ 2 = R 2 + a 2 cos 2 θ Δ = R 2 R S R + a 2

In der Gleichung J ist der Drehimpuls des Schwarzen Lochs, R ist die Entfernung vom Zentrum des Schwarzen Lochs, θ ist der Breitengrad, ϕ ist der Längengrad und T ist an der Zeit.

Dies vereinfacht, weil wir davon ausgehen können, dass alle Bewegungen in der Äquatorebene stattfinden θ ist konstant und daher D θ = 0 Und ρ = R . Wir betrachten auch den Moment, in dem sich das Licht tangential zu unserem Radiuskreis bewegt R , also an dieser Stelle D R 0 . Endlich für Licht D S = 0 , und all dies vereinfacht die Metrik zu:

0 = ( 1 R S R ) D T 2 + ( R 2 + a 2 + R S a 2 R ) D ϕ 2 + 2 R S a R D T D ϕ

Und schlussendlich D ϕ / D T ist nur die Winkelgeschwindigkeit ω also wenn wir durch dividieren durch D T 2 und umstellen erhalten wir:

(1) 0 = ( R 2 + a 2 + R S a 2 R ) ω 2 + 2 R S a R ω ( 1 R S R )

Und das ist die Gleichung, die wir brauchen. Lassen Sie uns einen schnellen Plausibilitätscheck machen und das nicht-rotierende Limit nehmen, dh set a = 0 . Gleichung (1) liefert uns sofort:

R ω = v = ± 1 R S R

Was ist die richtige Antwort! Wir erhalten zwei Werte für ω mit gleichen Größen und entgegengesetzten Vorzeichen, die den beiden Lichtstrahlen entsprechen, die in entgegengesetzte Richtungen gehen. Die beiden Strahlen haben also identische Geschwindigkeiten v = 1 R S / R (Beachten Sie, dass wir Einheiten verwenden, in denen C = 1 ). Die Lichtgeschwindigkeit wird um einen Faktor von reduziert 1 R S / R wegen der Zeitdilatation.

OK, also beruhigt, gehen wir zurück zu Gleichung (1) und lösen sie für eine Nicht-Null-Rotation, dh a 0 . Die Gleichung ist nur quadratisch und die quadratische Formel gibt uns:

ω = R S a R ( R 2 + a 2 + R S a 2 R ) ± ( R S a R ) 2 + ( R 2 + a 2 + R S a 2 R ) ( 1 R S R ) ( R 2 + a 2 + R S a 2 R )

Ich beabsichtige nicht, die Algebra weiter zu führen, aber es sollte jetzt sofort offensichtlich sein, dass unsere beiden Werte für die Winkelgeschwindigkeit nicht gleich sind. Sie unterscheiden sich durch:

ω + ω = 2 R S a R ( R 2 + a 2 + R S a 2 R )

Und dieser Unterschied ist der Effekt des Rahmenziehens.

Wohl nein. Hier ist meine Begründung:

Beachten Sie zunächst, dass Sie, wenn Sie über Frequenzverschiebungen in einer generischen allgemein-relativistischen Umgebung sprechen, Ihre Beobachter (Emitter und Absorber) sowie die Photonenbahn angeben müssen. Die verallgemeinerte Doppler-Verschiebung ist dann nichts anderes als die Differenz der Zeitkomponente des Photonenimpulses, gemessen in den beiden Referenzrahmen. Dies wird eine speziell-relativistische Dopplerverschiebung sowie eine gravitative und kosmologische Rotverschiebung ergeben (die phänomenologisch auf Zeitdilatation bzw. räumliche Ausdehnung zurückgeführt werden können).

Nun gibt es zwei verschiedene kreisförmige Photonenbahnen in der Äquatorialebene eines Kerr-Schwarzen Lochs (vgl. Wikipedia ), eine prograde und eine retrograde. Ein Beobachter, der an festen Boyer-Lindquist-Koordinaten entlang einer der Bahnen sitzt, wird immer dieselbe Photonenenergie messen, egal wie oft das Photon das Schwarze Loch umkreist hat.

Wenn Sie jedoch kompliziertere Trajektorien betrachten (z. B. wenn Sie mit einem beliebigen Radius beginnen und Photonen in entgegengesetzte Richtungen um das Loch herum schießen), ändert sich diese Geschichte im Allgemeinen - aber Sie werden aufgrund der ungleichen Positionen der Absorber nicht mehr Äpfel mit Äpfeln vergleichen .